Quantcast
  • Register
PhysicsOverflow is a next-generation academic platform for physicists and astronomers, including a community peer review system and a postgraduate-level discussion forum analogous to MathOverflow.

Welcome to PhysicsOverflow! PhysicsOverflow is an open platform for community peer review and graduate-level Physics discussion.

Please help promote PhysicsOverflow ads elsewhere if you like it.

News

PO is now at the Physics Department of Bielefeld University!

New printer friendly PO pages!

Migration to Bielefeld University was successful!

Please vote for this year's PhysicsOverflow ads!

Please do help out in categorising submissions. Submit a paper to PhysicsOverflow!

... see more

Tools for paper authors

Submit paper
Claim Paper Authorship

Tools for SE users

Search User
Reclaim SE Account
Request Account Merger
Nativise imported posts
Claim post (deleted users)
Import SE post

Users whose questions have been imported from Physics Stack Exchange, Theoretical Physics Stack Exchange, or any other Stack Exchange site are kindly requested to reclaim their account and not to register as a new user.

Public \(\beta\) tools

Report a bug with a feature
Request a new functionality
404 page design
Send feedback

Attributions

(propose a free ad)

Site Statistics

205 submissions , 163 unreviewed
5,047 questions , 2,200 unanswered
5,345 answers , 22,709 comments
1,470 users with positive rep
816 active unimported users
More ...

  Getting diffeomorphisms from boundary conditions in $AdS_3$

+ 2 like - 0 dislike
1813 views

As usual I'm asking a question about boundary conditions for AdS${}_3$, based on the thesis by Porfyriadis.

He is solving equations $\mathcal{L}_\xi g_{\mu\nu}$ for AdS${}_3$ metric, with a given boundary conditions that are basically a fall off conditions in $r$, since we're interested in the asymptotic behavior (as $r\to\infty$). I've finally gotten to the part where he, using the ansatz for diffeomorphism $\xi^\mu=\sum\limits_n \xi^\mu_n(t,\phi)r^n$ ($\mu=t,r,\phi$), gets a set of 6 equations for coefficients. I'll write the component of metric that was used to obtain the equation next to the equation, for clarification:

$$(tt)\qquad \xi^r_{n-1}+l^2\xi^t_{n,t}+\xi^t_{n-2,t}=0,\ n\ge 2$$ $$(tr)\qquad l^4(n+1)\xi^t_{n+1}-l^4\xi^r_{n,t}+3l^2(n-1)\xi^t_{n-1}+2(n-3)\xi^t_{n-3}=0,\ n\ge 3$$ $$(t\phi)\qquad l^2\xi^t_{n,\phi}+\xi^t_{n-2,\phi}-l^2\xi^\phi_{n-2,t}=0,\ n\ge 1$$ $$(rr)\qquad l^2(n+1)\xi^r_{n+1}+(n-2)\xi^r_{n-1}=0,\ n\ge 2$$ $$(r\phi)\qquad l^2(n+1)\xi^r_{n+1}+(n-3)\xi^\phi_{n-3}+l^2\xi^r_{n,\phi}=0,\ n\ge 3$$ $$(\phi\phi)\qquad \xi^r_{n-1}+\xi^\phi_{n-2,\phi}=0,\ n\ge 2$$

I got that, and I understand how the author got that. But how did he find the $\xi^r,\ \xi^t$, and $\xi^\phi$ I don't get :\

He said: for ($rr$) equation, using backwards induction, since for large $n$ the series for $\xi^r$ must truncate, we get that the components $\xi^r_{2m}=0,\ m\ge 1$, and $\xi^r_{2m+1}=0,\ m\ge 1$, so the most general form of $\xi^r$ is

$$\xi^r=\xi^r_1(t,\phi)r+\xi^r_0(t,\phi)+\mathcal{O}(r^{-1})\quad (\star)$$

How did he get that? I mean, I tried putting from n=10 towards n=2, and for n=2 I get

$$3l^2\xi^r_3+0\cdot\xi^r_1=0$$

And that means that for n=even my odd terms are 0, if $n\ge 2$?

I could kinda relate this to the fact that the $\xi^r=\sum\limits_n \xi^r_n r^n$, and for n=2 I'll get 0, so only n that are less than that will contribute, since I am making a expansion around infintiy. But I don't know if I'm right about this. And how did he make other equations? I tried using the same 'reasoning' but cannot get what he gets. In equations ($tr$) and ($r\phi$) he just drops off the $\xi^r_n$ terms. Why? :\ And then all of a sudden he gets

$$\xi^t=\xi^t_0(t,\phi)+\xi^t_{-1}(t,\phi)\frac{1}{r}+\mathcal{O}(r^{-2})$$ $$\xi^\phi=\xi^\phi_0(t,\phi)+\xi^\phi_{-1}(t,\phi)\frac{1}{r}+\mathcal{O}(r^{-2})$$

How?! :( I'm desperate :(

This post imported from StackExchange Physics at 2014-03-07 13:35 (UCT), posted by SE-user dingo_d
asked Sep 17, 2013 in Theoretical Physics by dingo_d (110 points) [ no revision ]

1 Answer

+ 1 like - 0 dislike

Alright so for this part you actually just need to look at three equations: $$(rr)\qquad l^2(n+1)\xi^r_{n+1}+(n-2)\xi^r_{n-1}=0,\ n\ge 2$$ $$(r\phi)\qquad l^2(n+1)\xi^r_{n+1}+(n-3)\xi^\phi_{n-3}+l^2\xi^r_{n,\phi}=0,\ n\ge 3$$ $$(tr)\qquad l^4(n+1)\xi^t_{n+1}-l^4\xi^r_{n,t}+3l^2(n-1)\xi^t_{n-1}+2(n-3)\xi^t_{n-3}=0,\ n\ge 3$$

Let's start with the first one: $$(rr)\qquad l^2(n+1)\xi^r_{n+1}+(n-2)\xi^r_{n-1}=0,\ n\ge 2.$$ The guy assumes that you don't keep having non-zero $\xi^\mu_n$ for arbitrarily large $n$. In other words, there must be some $N$ so that $\xi^\mu_m = 0$ for $m>N$. Now let's assume that this upper limit $N$ is just 10.

Then looking at the $(rr)$ equation for $n = 11$, we find that the left term on the LHS must be zero, because of the upper limit. Thus the right hand term, and thus $\xi^r_{10}$ must be zero. Similarly, the case $n=10$ tells us $\xi^r_9$ is zero.

Now we can consider $n=9$, we find again that the first term is zero, because we already established that $\xi^r_{10}$ is zero. We keep doing this all the way down to and including $n=3$. We find that $\xi^r_m$ is zero for $m\ge 2$, and so $\xi^r = \xi^r_1 r + \xi^r_0 + ...$ . We would have arrived at this conclusion regardless of how big $N$ was.

Ok good. What about the $(r\phi )$ equation? It is $$(r\phi)\qquad l^2(n+1)\xi^r_{n+1}+(n-3)\xi^\phi_{n-3}+l^2\xi^r_{n,\phi}=0,\ n\ge 3$$ We have already shown that $\xi^r_{n+1}$ and $\xi^r_{n}$ are zero for $n \ge 2$, and since this equation only applies to $n \ge 3$, we can drop these terms. The equation then becomes $$(r\phi)\qquad (n-3)\xi^\phi_{n-3}=0,\ n\ge 3$$ Now by considering this equation for $n \ge 4$, we find that $\xi^\phi_m=0$ for $m \ge 1$, and so $\xi^\phi = \xi^\phi_0 + \xi^\phi_{-1}/r + ...$ .

Finally let's consider the $(tr)$ equation:$$(tr)\qquad l^4(n+1)\xi^t_{n+1}-l^4\xi^r_{n,t}+3l^2(n-1)\xi^t_{n-1}+2(n-3)\xi^t_{n-3}=0,\ n\ge 3$$ Since the equation only applies for $n\ge 3$, and we know $\xi^r_{n}=0$ for $n\ge 3$, we can just ignore this term. The equation becomes $$(tr)\qquad l^4(n+1)\xi^t_{n+1}+3l^2(n-1)\xi^t_{n-1}+2(n-3)\xi^t_{n-3}=0,\ n\ge 3.$$ Let's again assume $N=10$, and consider the equation for $n=13$. Then the two leftmost terms disappear and we are left with $\xi^t_{10}=0$. Proceeding with $n=12$, $n=11$, all the way down to $n=4$ tells us that $\xi^t_m = 0$ for $m\ge 1$, and so $\xi^t = \xi^t_0 + \xi^t_{-1}/r + ...$ .

I think these were the parts you didn't get. Notice we didn't use three of the equations. The next thing the guy does is to milk some additional information out of those equations, but that wasn't your question I don't think. If you have questions about my answer be sure to ask.

This post imported from StackExchange Physics at 2014-03-07 13:35 (UCT), posted by SE-user NowIGetToLearnWhatAHeadIs
answered Sep 18, 2013 by NowIGetToLearnWhatAHeadIs (80 points) [ no revision ]
so I pick a number (say n=5), and input them in equations until I get an equation which will give me $0\cdot\xi^\mu_n$ where n is the number I got to from 5. Then I look at, what the coefficient is next to that $0\cdot\xi^\mu_n$ (in case of ($rr$) equation it was $\xi^r_1$, and in the case of ($r\phi$) equation it was $\xi^\phi_0$), and say that only nonzero contribution is from that coefficient lower in expansion? Am I understanding this right?

This post imported from StackExchange Physics at 2014-03-07 13:35 (UCT), posted by SE-user dingo_d
ya that is right.

This post imported from StackExchange Physics at 2014-03-07 13:35 (UCT), posted by SE-user NowIGetToLearnWhatAHeadIs
Thanks, you saved me again xD I need to understand how they get this so that I can use this in 4D case with near horizon extreme Kerr metric which is more complicated, and I have system of 10 equations like this. I hope I'll be able to reproduce their result. Thanks again :)

This post imported from StackExchange Physics at 2014-03-07 13:35 (UCT), posted by SE-user dingo_d

Your answer

Please use answers only to (at least partly) answer questions. To comment, discuss, or ask for clarification, leave a comment instead.
To mask links under text, please type your text, highlight it, and click the "link" button. You can then enter your link URL.
Please consult the FAQ for as to how to format your post.
This is the answer box; if you want to write a comment instead, please use the 'add comment' button.
Live preview (may slow down editor)   Preview
Your name to display (optional):
Privacy: Your email address will only be used for sending these notifications.
Anti-spam verification:
If you are a human please identify the position of the character covered by the symbol $\varnothing$ in the following word:
p$\hbar$y$\varnothing$icsOverflow
Then drag the red bullet below over the corresponding character of our banner. When you drop it there, the bullet changes to green (on slow internet connections after a few seconds).
Please complete the anti-spam verification




user contributions licensed under cc by-sa 3.0 with attribution required

Your rights
...